Deux intégrales pour cette fin de semaine — Les-mathematiques.net The most powerful custom community solution in the world

Deux intégrales pour cette fin de semaine

Je vous propose de calculer ces deux intégrales pour le plaisir du calcul:

$\displaystyle J=\int_1^{\infty} \dfrac{\ln(x+\sqrt{x^2-1)}}{x(1+x^2)}dx$

et:

$\displaystyle K=\int_0^{1} \dfrac{\arctan x}{\sqrt{1-x^2}}dx$

La seconde intégrale est plus facile à calculer que la première à mon humble avis.

Réponses

  • @FDP

    J'ai deux idées que je n'ai pas finalisé pour K ( edit lesdeux methodes marchent bien)
    1-noter $I(a)=\int_0^{1} \dfrac{\arctan ax}{\sqrt{1-x^2}}dx$
    apres calcul $I'(a)=\int_{0}^{1}\frac{x}{(1+x^2 a^2)\sqrt{1-x^2}}=\frac{1}{a\sqrt{1+a^2}}\,\text{arctanh}\frac{a}{\sqrt{1+a^2}}$ donc I(a)=......

    2-on se ramene par ipp à calculer $\int_0^{1} \dfrac{\arcsin x}{{1+x^2}}dx$ qui se rame à calculer $ M=\int_{0}^{\pi/2}\frac{x\cos x}{1+\sin^2x}\ \mathrm{d}x$ qui donne avec un peu de calcul $M=\frac12 \ln^2 (1+\sqrt{2})$
    Le 😄 Farceur


  • bonjour

    pour la première intégrale (dont la convergence est assurée)
    un changement de variable x = cht (cosinus hyperbolique) convient ; il en résulte l'intégrale :

    $\int_0^{+\infty}\frac{t.sht}{cht(1+ch^2t)}dt$ soit encore après décomposition

    $$\int_0^{+\infty}[t.tht - \frac{t.sht.cht}{1+ch^2t}]dt$$

    on termine par une intégration par parties

    cordialement
  • C'est sans doute la preuve la plus simple.

    Il y a d'autres preuves (au moins une mais j'imagine qu'on peut trouver des variantes) qui ne font pas appel aux fonctions hyperboliques.

    J'avais pensé à faire le même changement de variable mais j'ai enchaîné un peu trop vite vers un autre changement de variable qui donne l'intégrale:

    $\displaystyle \dfrac{1}{2}\int_0^1 \dfrac{x\ln\left(\dfrac{1+x}{1-x}\right)}{2-x^2}dx$

    Je pense qu'on peut exprimer cette intégrale comme somme de dilogarithmes de nombres réels et j'imagine qu'on peut simplifier cette somme par des formules usuelles.
    (j'essaierai de donner le calcul si c'est conforme à mes espérances)

    PS:
    Je n'avais pas vu que Gebrane avait affiné son esquisse. Cette façon de faire doit fonctionner.
  • @FDP
    L'idée marche parce que http://sos440.tistory.com/83

    Dans mes archives un exercice , démontrer que $\displaystyle \int_0^1 \dfrac{x\ln\left(\dfrac{1+x}{1-x}\right)}{2-x^2}dx=-\dfrac{\ln^2\left(2^\frac{3}{2}+3\right)+\ln^2\left(3-2^\frac{3}{2}\right)}{8}$
    Le 😄 Farceur


  • Bonjour,
    Je propose de montrer que
    $\displaystyle\int\limits^{1}_{0} \dfrac{\ln\left(x\right)\ln^2\left(1-x\right)}{x}\,\mathrm{d}x=-\frac{\zeta (4)}2(=-\frac {\pi^4}{180})$

    Wolfram https://www.wolframalpha.com/input/?i=\int_0^1+ln(x)ln^2(1-x)+/x
    Le 😄 Farceur


  • Les deux intégrales dans le premier message sont issues du problème E3140 de la revue American mathematical monthly. Je n'ai pas accès à sa correction dans le même journal.
    (on peut trouver sur le web un forum qui donne une solution)

    Je me demande s'il y a une façon de calculer la seconde à la manière dont Jean Lismonde a calculé la première.
  • Bonjour @gebrane0,

    Montrer que $\displaystyle \int_{0}^{1} {\ln(x) \ln^2(1-x) \over x} dx = -{\zeta(4) \over 2} = - {\pi^4 \over 180}.$

    Existence :
    L'intégrande est défini et continu sur $ \displaystyle ]0,1[$ et peut être prolongé par continuité en $0$ et $1$ car équivalent à la fonction $\displaystyle x \mapsto x \ln(x) \to 0, (x \to 0)$ en $0$ et à la fonction $\displaystyle x \mapsto -(1-x) \ln^2(1-x) \to 0, (x \to 1)$ en $1$ qui sont toutes les deux intégrables en $0$ et $1.$
    On a établi que $I$ existe.

    Changement de variables ou intégration par partie :
    On change la variable $\displaystyle x \leadsto y$ avec $\displaystyle y=1-x$, ou on intègre par partie, pour obtenir : $\displaystyle I = \int_{0}^{1} {\ln(1-x) \ln^2(x) \over 1-x} dx.$

    Equation différentielle et développement en série entière :
    La fonction $\displaystyle y: x \mapsto {\ln(1-x) \over 1-x}$ est définie, continue et dérivable sur $\displaystyle ]0,1[$ : on obtient l'équation différentielle dont $y$ est une solution : $\displaystyle (1-x)^2 y' - (1-x) y = -1, y(0)=0$ dont on cherche les solutions développables en série entière sous la forme $\displaystyle y(x) = \sum_{k \geq 0} a_k x^k.$
    On trouve facilement $\displaystyle -a_0+a_1=-1, -a_1+a_0+2a_2-2a_1=0$ et $\displaystyle k a_{k-1} - (2k+1)a_k + (k+1)a_{k+1} = 0, k \geq 0.$ On en déduit $\displaystyle a_0=0, a_1=-1, a_2 = -{3 \over 2}, a_k = -H_k, k \geq 2$ où $\displaystyle H_k$ est le nombre harmonique de rang $k.$ On démontre cette relation par récurrence.
    On a donc établi que $\displaystyle I=\int_{0}^{1} \sum_{k \geq 0} a_k x^k \ln^2(x) dx.$

    Changement de variables :
    On ne sait pas justifier l'inversion. Cependant, l'intégrale est immédiate avec le changement de variables $\displaystyle x \leadsto y$ avec $\displaystyle x^{k+1} = y$ et on trouve $\displaystyle \int_{0}^{1} \sum_{k \geq 0} {a_k \over (k+1)^3} \ln^2(y) dy = 2 \sum_{k \geq 0} {a_k \over (k+1)^3}.$ On a établi que $\displaystyle I =-2 \sum_{k \geq 1} {1 \over k^3} H_{k-1}.$
    Pour conclure, il vaut mieux savoir que $\displaystyle \sum_{k \geq 1} {1 \over k^3} H_{k} = \frac54 \zeta(4)$ : voir formule $(19)$ dans http://mathworld.wolfram.com/HarmonicNumber.html (*).
    Un soupçon d'arithmétique permet de conclure que $\displaystyle \sum_{k \geq 1} {1 \over k^3} H_{k-1} = \sum_{k \geq 1} {1 \over k^3} (H_k - {1 \over k}) = \frac14 \zeta(4).$ On a établi le résultat recherché.

    (*) Une formule plus générale, due à Euler, est : $\displaystyle 2 \sum_{n \geq 1} {H_n \over n^m} = (m+2) \zeta(m+1) - \sum_{n=1}^{m-2} \zeta(m-n)\zeta(n+1).$
  • YvesM:

    Merci de l'intérêt porté à la question malgré la chaleur qui ne donne pas envie de faire surchauffer les neurones. B-)-

    On établit directement, me semble-t-il, par l'utilisation du produit de Cauchy la formule bien connue pour $|x|<1$,

    $\displaystyle -\dfrac{\ln(1-x)}{1-x}=\sum_{n=1}^{\infty} H_nx^n$

    La partie intéressante du calcul, selon moi, est:

    $\displaystyle \sum_{k \geq 1} {1 \over k^3} H_{k} = \frac54 \zeta(4)$
  • Bonjour YvesM
    Tu es invincible .
    @FDP Le calcul de $\sum_{n \geq 1} \frac {H_n}{(n+1)^3}$ est simple si on sait que $H_n=\int_0^1 \frac{1-t^n}{1-t}dt$

    Voila une de monstrueuse qui peut vaincre YvesM et FDP :-)
    \begin{align*} I &= \int_{0}^{1} \frac{\log x \log (1 - x) \log^{2} (1 + x)}{x} \, dx. \end{align*}

    Rep: $I = \frac{7}{8}\zeta(2)\zeta(3) - \frac{25}{16} \zeta(5)$
    Le 😄 Farceur


  • Je reviens sur l'intégrale $\displaystyle J=\int_1^{\infty} \dfrac{\ln(x+\sqrt{x^2-1)}}{x(1+x^2)}dx$ car je n'arrive pas à terminer le calcul de Jean Lismonde. Je propose une méthode différente, par le théorème des résidus.

    On commence effectivement par poser $x=\mathrm{ch}(t)$, ce qui donne $\displaystyle 2J=\int_\R \frac{t\mathrm{sh}(t)}{\mathrm{ch} t(1+\mathrm{ch}^2(t))}\,dt$.

    Posons $\displaystyle f(t)= \frac{t^2\mathrm{sh}(t)}{\mathrm{ch} t(1+\mathrm{ch}^2(t))}$. En utilisant que $\displaystyle f(t)-f(t+i\pi)= \frac{(-2\pi i t+\pi^2)\mathrm{sh}(t)}{\mathrm{ch} t(1+\mathrm{ch}^2(t))}$, on trouve que
    $$\int_\R f(t)-f(t+i\pi)\,dt=-4\pi i J.$$

    On en déduit facilement, en intégrant sur le rectangle de sommets $\pm R$ et $\pm R+i\pi$ pour $R$ tendant vers l'infini, que $-2J=\sum_a \mathrm{Res}(f,a)$ où $a\in \{i\frac{\pi}{2},\pm\ln(1+\sqrt{2})+i\frac{\pi}{2}\}$.

    En utilisant la formule $ \mathrm{Res}(g/h,a)=g(a)/h'(a)$ pour $g(z)=z^2/(1+\mathrm{ch}^2(z))$ et $h(z)=\mathrm{ch}(z)$, on calcule que $\mathrm{Res}(f,i\frac{\pi}{2})=-\pi^2/4$.

    De même, on calcule que pour $\epsilon=\pm 1$ on a $\mathrm{Res}(f,\epsilon\ln(1+\sqrt{2})+i\frac{\pi}{2})=-\frac{1}{2}(\epsilon\ln(1+\sqrt{2})+i\frac{\pi}{2})^2$.

    La somme des résidus vaut donc $-\ln^2(1+\sqrt{2})$, et il vient :

    $$J=\frac{1}{2}\ln^2(1+\sqrt{2}).$$
  • Moi aussi, après approfondissement de la question, je ne sais pas comment Jean Lismonde termine son calcul.
    Je pensais que l'intégration par parties dont parle probablement Jean Lismonde allait de soit, on factorise l'intégrande $t\times Q(t)$
    On peut calculer une primitive de la fonction Q mais on va être en peine de trouver une primitive à cette primitive et par ailleurs, selon Wolfram, l'intégrale à considérer ne converge pas. :-D
    Il y a d'autres possibilité de procéder à une intégration par parties, mais j'ai l'impression que cela va compliquer le calcul sans apporter la solution attendue. B-)-

    PS:
    Cela dit on peut par le même changement de variable opéré par Jean Lismonde calculer cette intégrale sans faire appel à des calculs de résidu. C'est esquissé plus haut.
  • @JLT
    D’après ton calcul et le mien , on doit pouvoir démontrer que
    $\displaystyle \int_1^{\infty} \dfrac{\ln(x+\sqrt{x^2-1)}}{x(1+x^2)}dx =\int _0^{\pi /2}\frac{x\cos \:x}{1+\sin ^2x}dx$

    edit @FDP
    Je me souviendrai de vérifier d'abord sur MSE :-(
    Le 😄 Farceur


  • Gebrane0:
    Beaucoup d'intégrales "intéressantes" sont déjà traitées, soit sur M.E, soit sur des forums que tu connais très probablement. Un certain nombre sont issues de concours anglo-saxons connus (Putnam) et de problèmes posés dans des revues.
    Cela dit, il n'est pas inintéressant de donner d'autres méthodes de résolution.
  • Ci-après le calcul élémentaire de:

    $\displaystyle J=\int_1^{\infty} \dfrac{\ln(x+\sqrt{x^2-1)}}{x(1+x^2)}dx$

    Comme Jean Lismonde faisons le changement de variable $x=\text{ch} t$

    $\begin{align}J&=\int_0^{\infty} \dfrac{t\text{sh} t}{\text{ch} t(1+\text{ch}^2 t)}dt\\
    &=\int_0^{\infty} \dfrac{t\text{th} t}{1+\text{ch}^2 t}dt\\
    &=\int_0^{\infty} \frac{t\text{th} t}{\text{ch}^2 t\left(\frac{1}{\text{ch}^2 t}+1\right)}dt\\
    &=\int_0^{\infty} \frac{t\text{th} t}{\text{ch}^2 t\left(1-\text{th}^2t+1\right)}dt\\
    &=\int_0^{\infty} \frac{t\text{th} t}{\text{ch}^2 t\left(2-\text{th}^2t\right)}dt\\
    \end{align}$

    On fait le changement de variable $x=\text{th}t$,

    $\begin{align}J&=\int_0^1 \dfrac{x\text{arctanh} x}{2-x^2}dx\\
    &=\dfrac{1}{2}\int_0^1 \frac{x\ln\left(\frac{1+x}{1-x}\right)}{2-x^2}dx\\
    &=\dfrac{1}{4}\int_0^1 \frac{\ln\left(\frac{1-x}{1+x}\right)}{x-\sqrt{2}}dx+\dfrac{1}{4}\int_0^1 \frac{\ln\left(\frac{1-x}{1+x}\right)}{x+\sqrt{2}}dx
    \end{align}$

    On fait le changement de variable $y=\dfrac{1-x}{1+x}$,

    $\begin{align}J&=-\frac{1}{2}\int_0^1 \frac{\ln x}{(1+x)\Big((\sqrt{2}+1)x+\sqrt{2}-1\Big)}dx+\frac{1}{2}\int_0^1 \frac{\ln x}{(1+x)\Big((\sqrt{2}-1)x+\sqrt{2}+1\Big)}dx
    \end{align}$

    Par ailleurs,

    $\alpha=\dfrac{\sqrt{2}+1}{\sqrt{2}-1}=(1+\sqrt{2})^2$,$\beta=\dfrac{\sqrt{2}-1}{\sqrt{2}+1}$ et $\alpha\beta=1$


    $\begin{align}J&=\frac{1}{4}\left(\int_0^1 \dfrac{\ln x}{x+1}dx-\beta\int_0^1 \dfrac{\ln x}{\beta x+1}dx\right)+\frac{1}{4}\left(\int_0^1 \dfrac{\ln x}{x+1}dx-\alpha\int_0^1 \dfrac{\ln x}{\alpha x+1}dx\right)\\
    &=\frac{1}{2}\int_0^1 \dfrac{\ln x}{x+1}dx-\frac{1}{4}\left(\beta\int_0^1 \dfrac{\ln x}{\beta x+1}dx+\alpha\int_0^1 \dfrac{\ln x}{\alpha x+1}dx\right)
    \end{align}$

    Dans la deuxième intégrale on fait le changement de variable $y=\beta x$,
    Dans la troisième intégrale on fait le changement de variable $y=\alpha x$,

    $\begin{align}J&=\frac{1}{2}\int_0^1 \dfrac{\ln x}{x+1}dx-\frac{1}{4}\left(\int_0^{\beta} \dfrac{\ln(\alpha x)}{x+1}dx+\int_0^{\alpha} \dfrac{\ln(\beta x)}{x+1}dx \right)\\
    &=\frac{1}{2}\int_0^1 \dfrac{\ln x}{x+1}dx-\frac{1}{4}\left(\int_0^{\beta} \dfrac{\ln x}{x+1}dx+\int_0^{\alpha} \dfrac{\ln x}{x+1}dx \right)-\frac{1}{4}\left(\int_0^{\beta} \dfrac{\ln \alpha}{x+1}dx+\int_0^{\alpha} \dfrac{\ln \beta }{x+1}dx \right)\\
    &=\frac{1}{2}\int_0^1 \dfrac{\ln x}{x+1}dx-\frac{1}{4}\left(\int_0^{\beta} \dfrac{\ln x}{x+1}dx+\int_0^{\alpha} \dfrac{\ln x}{x+1}dx \right)-\frac{1}{4}\big(\ln \alpha \ln(1+\beta)+\ln \beta\ln(1+\alpha)\big)\\
    &=\frac{1}{2}\int_0^1 \dfrac{\ln x}{x+1}dx-\frac{1}{4}\left(\int_0^{1} \dfrac{\ln x}{x+1}dx+\int_1^{\beta} \dfrac{\ln x}{x+1}dx+\int_0^{1} \dfrac{\ln x}{x+1}dx+\int_1^{\alpha} \dfrac{\ln x}{x+1}dx\right)+\frac{1}{4}\ln^2 \alpha\\
    &=-\frac{1}{4}\left(\int_1^{\beta} \dfrac{\ln x}{x+1}dx+\int_1^{\alpha} \dfrac{\ln x}{x+1}dx\right)+\frac{1}{4}\ln^2 \alpha\\
    \end{align}$

    Dans la première intégrale on effectue le changement de variable $y=\dfrac{1}{x}$,

    $\begin{align}J&=-\frac{1}{4}\left(\int_1^{\alpha} \dfrac{\ln x}{x(x+1)}dx+\int_1^{\alpha} \dfrac{\ln x}{x+1}dx\right)+\frac{1}{4}\ln^2 \alpha\\
    &=-\frac{1}{4} \int_1^{\alpha} \left(\dfrac{\ln x}{x(1+x)}+\dfrac{\ln x}{1+x}\right)dx+\frac{1}{4}\ln^2 \alpha\\
    &=-\frac{1}{4} \int_1^{\alpha} \dfrac{\ln x}{x}dx+\frac{1}{4}\ln^2 \alpha\\
    &=-\frac{1}{8}\Big[\ln^2 x\Big]_1^{\alpha}+\frac{1}{4}\ln^2 \alpha\\
    &=\frac{1}{8}\ln^2 \alpha\\
    &=\frac{1}{8}\ln^2\left((1+\sqrt{2})^2\right)\\
    &=\boxed{\frac{1}{2}\ln(\sqrt{2}+1)^2}
    \end{align}$

    Edit:
    J'ai simplifié le calcul qui donne le résultat sous la forme attendue pour éviter de tourner en rond. Je développais initialement $(1+\sqrt{2})^2$ pour terminer par, en substance, me rendre compte à la fin que ce nombre était le carré de $1+\sqrt{2}$ :-D
  • Pendant mes "vacances" j'ai cherché, en vain, un moyen de déduire K de J en utilisant seulement comme outil, le changement de variable, l'intégration par parties, sans utiliser d'intégrales doubles, d'intégrales à paramètre.

    C'est sans doute lié au fait qu'on ne sait pas montrer que l'intégrale $\displaystyle \int_ 0^1 \frac{\ln x}{x-1}dx=\frac{\pi^2}{6}$ en se restreignant aux mêmes outils.

    Quoiqu'il en soit je vous propose pour cette fin de semaine les deux intégrales.

    $\displaystyle J=\int_0^{\frac{\pi}{2}} \frac{\sin x}{1+\sqrt{\sin(2x)}}dx$

    $\displaystyle K=\int_0^{\frac{\pi}{2}} \frac{x^2}{\sin x}dx$

    La deuxième intégrale est, à mon humble avis, la plus compliquée des deux à calculer.
  • @FDP
    Au debut de ton récit, tu parlais de quels K et J ( les anciens ou les nouveaux?)
    Le 😄 Farceur


  • Les "anciennes" valeurs.

    J'ai regardé la solution proposée par American mathematical monthly ils ne font pas "mieux" que moi.
    L'une des intégrales est calculée directement par changement de variable (comme je l'ai fait) et l'autre est calculée en passant par une intégrale double ou en introduisant une intégrale à paramètre.

    PS:
    Il faudrait que je m'occupe de ton intégrale à l'occasion. ;-)
  • $J=\dfrac{\pi}2-1$ et $K=2\pi Catalan-\dfrac72\zeta(3)$.
  • Jandri:

    Je suis intéressé par les calculs.
    Cordialement.
  • Un logiciel me donne $K=2\operatorname{Li}_3\left(\mathrm{i}\right)-\mathrm{i}{\pi}\operatorname{Li}_2\left(\mathrm{i}\right)-2\operatorname{Li}_3\left(-\mathrm{i}\right)+\mathrm{i}{\pi}\operatorname{Li}_2\left(-\mathrm{i}\right)-\dfrac{7\operatorname{\zeta}\left(3\right)}{2}-\dfrac{\mathrm{i}{\pi}^3}{8}=1.547982402157742...$
    Le 😄 Farceur


  • Le calcul de $J$ n'est pas compliqué.

    J'ai posé $x=\dfrac{\pi}4+t$ puis $\sqrt2\sin(t)=\sin(\theta)$.
  • Gebrane0:

    Wolfram donne la valeur de l'intégrale K (la nouvelle). C'est la valeur donnée par Jandri.

    L'intégrale J est un problème donné très récemment par American mathematical monthly.
  • .
    Le 😄 Farceur


  • D'abord je ne vois pas pourquoi l'on s'interdirait les intégrales ou à paramètres ou les intégrales doubles, qui sont tout de même élémentaires, même si le législateur a supprimé ces dernières du programme de math. spé. en 2013.

    Pour $\displaystyle \int_ 0^1 \frac{\ln x}{x-1}dx=\frac{\pi^2}{6}$, il est bien connu que la méthode est le développement en série entière. Comme il s'agit d'une série géométrique, le reste s'exprime explicitement et on peut le faire sans aucune connaissance sur les séries entières, niveau prépa-HEC.

    Bonne après-midi.
    Fr. Ch.
  • Chaurien:

    Je pense que tu es en train de m'expliquer qu'on peut facilement obtenir que:

    $\displaystyle \int_ 0^1 \frac{\ln x}{x-1}dx=\zeta(2)$


    Mais explique moi comment, avec les seuls outils que j'ai indiqués plus haut, tu montres que $\zeta(2)=\frac{\pi^2}{6}$.
    Je suis très intéressé.
    Cordialement.

    PS:
    On peut utiliser seulement du calcul sur les séries et du calcul intégral très élémentaire pour montrer cette égalité.
    C'est bien pour cela que ma question portait sur l'égalité entre une intégrale et $\frac{\pi^2}{6}$.
    Il s'agit d'utiliser seulement l'intégration par partie et le changement de variable sur des intégrales non multiples.


    PS2:
    Pour rappel,
    http://www.les-mathematiques.net/phorum/read.php?4,1400462
  • L'intégrale,

    $\displaystyle \int_0^1 \frac{\ln^2 x\ln(1-x)}{1-x}dx$

    est égale à $\displaystyle \frac{\partial^3}{\partial^2 r\partial s}\text{B}(r,s)_{r=1,s=0}$

    avec $\text{B}$, la fonction Beta d'Euler.

    Ce qui fait que si on connait les valeurs $\Gamma^{\prime}(0)$,$\Gamma^{\prime}(1),...$
    On peut calculer cette intégrale. Le calcul est pénible cela dit.
  • Généralement, on prouve comme tu dis que $\displaystyle \int_ 0^1 \frac{\ln x}{x-1}dx=\zeta(2)$.
    Par ailleurs, il y a une foule de démonstrations de $\zeta(2)=\frac{\pi^2}{6}$.
    Dans le fil que tu rappelles il y en a déjà pas mal, la plupart venant de toi.
    Les deux se rejoignent dans la démonstration avec l'intégrale double.
    Bonne soirée.
    Fr. Ch.
  • Soit $K=\displaystyle\int_0^{\pi/2}\dfrac{x^2}{\sin x}\,dx$. Voici une justification de l'égalité trouvée par jandri : $K=2\pi G-\frac{7}{2}\zeta(3)$, où $G=\displaystyle\sum_{k=0}^\infty \dfrac{(-1)^k}{(2k+1)^2}$.

    Soit $f(x)$ la fonction impaire $2\pi$-périodique telle que $f(x)=x^2$ pour $x\in [0,\frac{\pi}{2}]$ et $f(x)=(\pi-x)^2$ pour $x\in [\frac{\pi}{2},\pi]$. En calculant les coefficients de Fourier de $f$, on obtient $f(x)=\displaystyle\sum_{k=0}^\infty b_k\sin(2k+1)x$ où $b_k=\dfrac{4(-1)^k}{(2k+1)^2}-\dfrac{8}{\pi(2k+1)^3}$.

    Or, on a $\displaystyle\int_0^{\pi/2}\frac{\sin(2k+1)x}{\sin x}\,dx=\frac{\pi}{2}$ (utiliser $\frac{\sin(2k+1)x}{\sin x}=\displaystyle\sum_{j=-k}^{k}e^{2ijx}$), donc $K=\displaystyle\sum_{k=0}^\infty \dfrac{2\pi(-1)^k}{(2k+1)^2}-\frac{4}{(2k+1)^3}$.

    Édit : ajout d'un $(-1)^k$ oublié à la dernière ligne, merci jandri.
  • Une alternative pour le calcul de ,

    $\displaystyle A=\int_0^1 \frac{\ln^2 x\ln(1-x)}{1-x}dx$

    On considère l'intégrale,

    $\displaystyle B=\int_0^1 \frac{\ln x\ln^2\left( \frac{x}{1-x}\right)}{1-x}dx \tag 1$

    On a,

    $\begin{align}
    B&=\int_0^1 \frac{\ln x\big(\ln x-\ln(1-x)\big)^2}{1-x}dx\\
    &=\int_0^1 \frac{\ln^3 x}{1-x}dx-2A+\int_0^1 \frac{\ln x\ln^2(1-x)}{1-x}dx
    \end{align}$

    Dans la dernière intégrale on fait le changement de variable $y=1-x$,

    $\displaystyle B=\int_0^1 \frac{\ln^3 x}{1-x}dx-2A+\int_0^1 \frac{\ln(1-x)\ln^2 x}{x}dx$

    Par ailleurs, si on fait le changement de variable $y=\frac{x}{1-x}$ dans (1),

    $\begin{align}
    B&=\int_0^{\infty} \frac{\ln^3 x-\ln^2 x\ln(1+x)}{1+x}dx\\
    &=\int_0^{1} \frac{\ln^3 x-\ln^2 x\ln(1+x)}{1+x}dx+\int_1^{\infty} \frac{\ln^3 x-\ln^2 x\ln(1+x)}{1+x}dx\\
    &=\int_0^{1} \frac{\ln^3 x}{1+x}dx-\int_0^{1} \frac{\ln^2 x\ln(1+x)}{1+x}dx+\int_1^{\infty} \frac{\ln^3 x-\ln^2 x\ln(1+x)}{1+x}dx
    \end{align}$

    Dans la dernière intégrale on effectue le changement de variable $y=\frac{1}{x}$,
    $\begin{align}B&=\int_0^{1} \frac{\ln^3 x}{1+x}dx-\int_0^{1} \frac{\ln^2 x\ln(1+x)}{1+x}dx-\int_0^{1} \frac{\ln^2 x\ln(1+x)}{x(1+x)}dx\\
    &=\int_0^{1} \frac{\ln^3 x}{1+x}dx-\int_0^{1} \frac{\ln^2 x\ln(1+x)}{1+x}dx-\Big(\int_0^1\frac{\ln^2 x\ln(1+x) }{x}dx-\int_0^1\frac{\ln^2 x\ln(1+x)}{1+x}dx\Big)\\
    &=\int_0^{1} \frac{\ln^3 x}{1+x}dx-\int_0^1\frac{\ln^2 x\ln(1+x) }{x}dx
    \end{align}$

    Ce qui fait qu'on a,

    $\displaystyle A=\frac{1}{2}\int_0^1 \frac{\ln^3 x}{1-x}dx-\frac{1}{2}\int_0^1 \frac{\ln^3 x}{1+x}dx+\frac{1}{2}\int_0^1 \frac{\ln(1-x)\ln^2 x}{x}dx+\frac{1}{2}\int_0^1 \frac{\ln(1+x)\ln^2 x}{x}dx$

    En développant les intégrandes de toutes ces intégrales en série de Taylor on peut obtenir qu'elles sont égales à un nombre rationnel fois $\zeta(4)$.

    Addendum:
    $\begin{align}\int_0^1 \frac{\ln(1+x)\ln^2 x}{x}dx&=\Big[\ln^3 x\ln(1+x)\Big]_0^1-\int_0^1\left(\frac{\ln^3 x}{1+x}+\frac{2\ln^2 x\ln(1+x)}{x}\right)dx\\
    &=-\int_0^1 \frac{\ln^3 x}{1+x}dx-2\int_0^1\frac{\ln^2 x\ln(1+x)}{x}dx
    \end{align}$

    Donc,
    $\displaystyle \int_0^1\frac{\ln^2 x\ln(1+x)}{x}dx=-\frac{1}{3}\int_0^1 \frac{\ln^3 x}{1+x}dx$

    $\begin{align}\int_0^1 \frac{\ln(1-x)\ln^2 x}{x}dx&=\Big[\ln^3 x\ln(1-x)\Big]_0^1-\int_0^1\left(\frac{2\ln^2 x\ln(1-x)}{x}-\frac{\ln^3 x}{1-x}\right)dx\\
    &=\int_0^1 \frac{\ln^3 x}{1-x}dx-2\int_0^1\frac{\ln^2 x\ln(1-x)}{x}dx
    \end{align}$

    Donc,
    $\displaystyle \int_0^1\frac{\ln^2 x\ln(1-x)}{x}dx=\frac{1}{3}\int_0^1 \frac{\ln^3 x}{1-x}dx$

    Ainsi,

    $\boxed{\displaystyle A=\frac{2}{3}\int_0^1 \frac{\ln^3 x}{1-x}dx-\frac{2}{3}\int_0^1 \frac{\ln^3 x}{1+x}dx}$

    Addendum 2:

    $\begin{align}
    A&=\frac{2}{3}\int_0^1 \frac{2x\ln^3 x}{1-x^2}dx\\
    &=\frac{2}{3}\times \frac{1}{8}\int_0^1 \frac{2x\ln^3(x^2)}{1-x^2}dx\\
    \end{align}$

    On fait le changement de variable $y=x^2$,


    $\boxed{\displaystyle A=\frac{1}{12}\int_0^1 \frac{\ln^3 x}{1-x}dx}$


    PS:
    Chaurien:
    Aucune de ces démonstrations ne se limitent aux outils que j'ai indiqués. Je crois en avoir vu un certain nombre.

    PS2:
    Merci JLT de l'intérêt porté à la question.
  • Bravo à JLT, c'est une très belle idée d'introduire une série de Fourier, je n'y avais pas pensé.
    Il a une coquille, il manque un $(-1)^k$ dans la première fraction de la dernière égalité.
  • $\displaystyle K=\int_0^{\frac{\pi}{2}} \frac{x^2}{\sin x}dx$

    Puisque,

    $\begin{align}\int \frac{1}{\sin x}dx&=\text{arctanh}(\cos x)dx+\text{constante}\\
    &=\frac{1}{2}\ln\left(\frac{1+\cos x}{1-\cos x}\right)+\text{constante}\\
    &=\ln\left(\tan\left(\frac{x}{2}\right)\right)+\text{constante}
    \end{align}$

    donc,

    $\displaystyle K=2\Bigg[\ln\left(\tan\left(\frac{x}{2}\right)\right)x\Bigg]_0^{\frac{\pi}{2}}-2\int_0^{\frac{\pi}{2}} \ln\left(\tan\left(\frac{x}{2}\right)\right)x\, dx$

    On fait le changement de variable $y=\frac{x}{2}$,

    $\displaystyle K=-8\int_0^{\frac{\pi}{4}} \ln\left(\tan\left(x\right)\right)x\, dx$

    On fait le changement de variable $y=\tan x$,

    $\displaystyle K=-8\int_0^1 \frac{\arctan x\ln x}{1+x^2}\, dx$

    On définit sur $[0;1]$ la fonction $R$,

    $\displaystyle R(x)=\int_0^x \frac{\ln t}{1+t^2}dt=\int_0^1 \frac{x\ln(tx)}{1+t^2x^2}dt$

    On a $R(0)=0$ et $R(1)=-\text{G}$.

    $\begin{align}\int_0^1 \frac{\arctan x\ln x}{1+x^2}dx&=\Big[R(x)\arctan x\Big]_0^1-\int_0^1 \frac{R(x)}{1+x^2}dx\\
    &=-\frac{1}{4}\pi G-\int_0^1 \int_0^1 \frac{x\ln(tx)}{(1+x^2)(1+t^2x^2)}\,dtdx\\
    &=-\frac{1}{4}\pi G-\int_0^1 \int_0^1 \frac{x\ln t }{(1+x^2)(1+t^2x^2)}\,dtdx-\int_0^1 \int_0^1 \frac{x\ln x}{(1+x^2)(1+t^2x^2)}\,dtdx\\
    &=-\frac{1}{4}\pi G+\frac{1}{2}\int_0^1 \left[\frac{\ln t}{1-t^2}\ln\left(\frac{1+t^2x^2}{1+x^2}\right)\right]_{x=0}^{x=1} dt-\int_0^1 \left[\frac{\ln x\arctan(tx)}{1+x^2}\right]_{t=0}^{t=1}\\
    &=-\frac{1}{4}\pi G+\frac{1}{2}\int_0^1 \frac{\ln t}{1-t^2}\ln\left(\frac{1+t^2}{2}\right)dt-\int_0^1 \frac{\arctan x\ln x}{1+x^2}dx
    \end{align}$

    Ainsi,

    $\begin{align} \int_0^1 \frac{\arctan x\ln x}{1+x^2}dx&=
    -\frac{1}{8}\pi G+\frac{1}{4}\int_0^1 \frac{\ln(1+t^2)\ln t}{1-t^2}dt-\frac{1}{4}\ln 2\int_0^1 \frac{\ln t}{1-t^2}dt\\
    &=-\frac{1}{8}\pi G+\frac{1}{4}\int_0^1 \frac{\ln(1+t^2)\ln t}{1-t^2}dt+\frac{1}{32}\pi^2\ln 2
    \end{align}$

    On définit sur $[0;1]$ la fonction $T$,

    $\displaystyle T(x)=\int_0^x \frac{\ln t}{1-t^2}dt=\int_0^1 \frac{x\ln(tx)}{1-t^2x^2}dt$

    On a $T(0)=0$ et $T(1)=-\frac{1}{8}\pi^2$.

    $\begin{align}
    \int_0^1 \frac{\ln(1+x^2)\ln x}{1-x^2}dx&=\Big[T(x)\ln(1+x^2)\Big]_0^1-2\int_0^1 \frac{xT(x)}{1+x^2} dx\\
    &=-\frac{1}{8}\pi^2\ln 2-2\int_0^1 \int_0^1 \frac{x^2\ln(tx)}{(1-t^2x^2)(1+x^2)}dtdx\\
    &=-\frac{1}{8}\pi^2\ln 2-2\int_0^1 \int_0^1 \frac{x^2\ln t}{(1-t^2x^2)(1+x^2)}dtdx-2\int_0^1 \int_0^1 \frac{x^2\ln x}{(1-t^2x^2)(1+x^2)}dtdx\\
    &=-\frac{1}{8}\pi^2\ln 2+\int_0^1 \left[\frac{\ln t}{t(1+t^2)}\ln\left(\frac{1-tx}{1+tx}\right)+\frac{2\ln t\arctan x}{1+t^2}\right]_{x=0}^{x=1}dt+\int_0^1 \left[\frac{x\ln x}{1+x^2}\ln\left(\frac{1-tx}{1+tx}\right)\right]_{t=0}^{t=1}dx\\
    &=-\frac{1}{8}\pi^2\ln 2+\int_0^1\frac{\ln t}{t(1+t^2)}\ln\left(\frac{1-x}{1+x}\right)dt+\frac{1}{2}\pi\int_0^1 \frac{\ln t}{1+t^2}dt+\int_0^1 \frac{x\ln x}{1+x^2}\ln\left(\frac{1-x}{1+x}\right)dx\\
    &=-\frac{1}{2}\text{G}\pi-\frac{1}{8}\pi^2\ln 2+\int_0^1 \frac{\ln x}{x}\ln\left(\frac{1-x}{1+x}\right)dx\\
    \end{align}$

    Par ailleurs,

    $\begin{align}
    \int_0^1 \frac{\ln x}{x}\ln\left(1-x\right)dx+\int_0^1 \frac{\ln x}{x}\ln\left(1+x\right)dx&=\int_0^1 \frac{\ln x}{x}\ln\left(1-x^2\right)dx\\
    &=\frac{1}{4}\int_0^1 \frac{2x\ln\left(x^2\right)}{x^2}\ln\left(1-x^2\right)dx\\
    \end{align}$

    On effectue le changement de variable $y=2x$,

    $\begin{align}
    \int_0^1 \frac{\ln x}{x}\ln\left(1-x\right)dx+\int_0^1 \frac{\ln x}{x}\ln\left(1+x\right)dx&=\frac{1}{4}\int_0^1 \frac{\ln x}{x}\ln\left(1-x\right)dx\\
    \end{align}$

    Ainsi,

    $\begin{align}
    \int_0^1 \frac{\ln(1+x^2)\ln x}{1-x^2}dx&=-\frac{1}{2}\text{G}\pi-\frac{1}{8}\pi^2\ln 2+\frac{7}{4}\int_0^1 \frac{\ln x}{x}\ln\left(1-x\right)dx\\
    \int_0^1 \frac{\arctan x\ln x}{1+x^2}dx&=-\frac{1}{4}\text{G}\pi+\frac{7}{16}\int_0^1 \frac{\ln x}{x}\ln\left(1-x\right)dx\\
    \end {align}$

    et,
    $\displaystyle K=2\text{G}\pi-\frac{7}{2}\int_0^1 \frac{\ln x}{x}\ln\left(1-x\right)dx$

    En développant l'intégrande de la dernière intégrale en série de Taylor et en intégrant terme à terme la série obtenue on obtient qu'elle est égale à $\zeta(3)$ et ainsi,

    $\boxed{\displaystyle K=2\text{G}\pi-\frac{7}{2}\zeta(3)}$
  • Dans un autre fil j'avais proposé cette intégrale à calculer,

    $\displaystyle \int_{0}^{\infty} \frac{1-x^2}{x(1+x^2)}\arctan\left(\frac{x}{1+x^2}\right)\ln x \,dx$

    Je la reposte parce qu'elle a un lien très étroit avec ce qui a été fait dans ce fil.
  • Une petite intégrale pas trop compliquée pour commencer cette longue fin de semaine.

    $\displaystyle \int_0^{\frac{3-\sqrt{5}}{2}}\frac{1+x}{\sqrt{1+x^2}(1-x+x^2)}dx$
  • Le changement de variable $x=\dfrac{1-y}{1+y}$ donne :
    $$\int_0^{(3-\sqrt{5})/2}\frac{1+x}{\sqrt{1+x^2}(1-x+x^2)}dx=2\sqrt{2}\int_{1/\sqrt{5}}^1\dfrac{dy}{\sqrt{1+y^2}(1+3y^2)}.$$
    Le changement de variable $y=\tan t$ donne :
    $$\int_{1/\sqrt{5}}^1\dfrac{dy}{\sqrt{1+y^2}(1+3y^2)}=\int_{\arctan(1/\sqrt{5})}^{\pi/4}\dfrac{\cos t}{1+2\sin^2t}dt.$$
    Enfin le changement de variable $u=\sin t$ donne :
    $$\int_{\arctan(1/\sqrt{5})}^{\pi/4}\dfrac{\cos t}{1+2\sin^2t}dt=\int_{1/\sqrt{6}}^{1/\sqrt{2}}\dfrac{du}{1+2u^2}=\left[\frac{\arctan(\sqrt{2}u)}{\sqrt{2}}\right]_{1/\sqrt{6}}^{1/\sqrt{2}}=\frac{\pi}{12\sqrt{2}}$$
    d'où finalement :
    $$\int_0^{(3-\sqrt{5})/2}\frac{1+x}{\sqrt{1+x^2}(1-x+x^2)}dx=\frac{\pi}{6}.$$
  • Bravo !

    En fait, j'ai fabriqué cette intégrale à partir du constat que,

    $\displaystyle \arctan\left(\frac{1+x}{\sqrt{1+x^2}}\right)$ est une primitive de $\displaystyle \frac{1-x}{2\sqrt{1+x^2}(1+x+x^2)}$

    J'ai rencontré cette dernière fonction dans un calcul que je faisais. J'ai été ennuyé de voir que Wolfram en ligne ne donnait pas de forme simple à une primitive de cette fonction.

    On a les changements de variable dits de Weierstrass qui permettent d'exprimer les fonctions cosinus et sinus comme fractions rationnelles de $\tan\left(\frac{x}{2}\right)$.
    Mais il y a des changements de variables qui permettent d'exprimer cosinus et sinus en fonction de $\tan x$
    (si $x$ appartient à $\left[0;\frac{\pi}{2}\right ]$)

    $\cos x=\frac{1}{\sqrt{1+x^2}}$ et $\sin x=\frac{x}{\sqrt{1+x^2}}$

    et,

    $\begin{align} \frac{1-x}{2\sqrt{1+x^2}(1+x+x^2)}&=\frac{1-x}{2\sqrt{1+x^2}(1+x^2)\left(1+\frac{x}{1+x^2}\right)}\\
    &=\frac{1}{2(1+x^2)}\times \frac{1}{1+\frac{x}{\sqrt{1+x^2}}\times \frac{1}{\sqrt{1+x^2}}}\times \left(\frac{1}{\sqrt{1+x^2}}-\frac{x}{\sqrt{1+x^2}}\right)
    \end{align}$

    Ce qui fait que lorsqu'on fait le changement de variable $x=\tan t$ dans l'intégrale,

    $\begin{align} \int \frac{1-x}{2\sqrt{1+x^2}(1+x+x^2)}dx&=\dfrac{1}{2}\int \frac{\cos t-\sin t}{1+\cos t\sin t}dt\\
    &=\int \frac{\cos t-\sin t}{2+2\cos t\sin t}dt\\
    &=\int \frac{\cos t-\sin t}{2+(\sin t+\cos t)^2-\cos^2 t-\sin^2 t}dt\\
    &=\int \frac{\cos t-\sin t}{1+(\sin t+\cos t)^2}dt\\
    &=\arctan(\sin t+\cos t)
    \end{align}$
  • J'ai trouvé une manière de calculer ton intégrale précédente http://www.les-mathematiques.net/phorum/read.php?4,1481230,1494196#msg-1494196 :
    $$I=\int_{0}^{\infty} \frac{1-x^2}{x(1+x^2)}\arctan\left(\frac{x}{1+x^2}\right)\ln x \,dx.$$
    Je pose $y=\dfrac{x}{1+x^2}$ en travaillant séparément sur $[0,1]$ et $[1,+\infty[$. J'obtiens :
    $$\int_{0}^{1} \frac{1-x^2}{x(1+x^2)}\arctan\left(\frac{x}{1+x^2}\right)\ln x \,dx=\int_0^{1/2}\frac{\arctan y}{y}\ln\frac{1-\sqrt{1-4y^2}}{2y}dy$$
    et
    $$\int_{1}^{+\infty} \frac{1-x^2}{x(1+x^2)}\arctan\left(\frac{x}{1+x^2}\right)\ln x \,dx=-\int_0^{1/2}\frac{\arctan y}{y}\ln\frac{1+\sqrt{1-4y^2}}{2y}dy$$
    ce qui donne
    $$I=\int_0^{1/2}\frac{\arctan y}{y}\ln\frac{1-\sqrt{1-4y^2}}{1+\sqrt{1-4y^2}}dy.$$
    J'intègre par parties et j'obtiens :
    $$I=-2\int_0^{1/2}\dfrac{\phi(y)}{y\sqrt{1-4y^2}}dy$$
    où $\phi(y)=\displaystyle\int_0^y\frac{\arctan t}{t}dt$.

    Je développe $\phi$ en série entière en 0 et j'intervertis $\sum$ et $\int$ :
    $$I=-2\sum_{n=0}^{+\infty}\dfrac{(-1)^n}{(2n+1)^2}\int_0^{1/2}\dfrac{y^{2n}}{\sqrt{1-4y^2}}dy.$$
    Le changement de variable $y=(\sin t)/2$ fait apparaître les intégrales de Wallis :
    $$\int_0^{1/2}\dfrac{y^{2n}}{\sqrt{1-4y^2}}dy=\dfrac{1}{2^{2n+1}}\int_0^{\pi/2}\sin^{2n}t\,dt=\dfrac{\pi}{2^{4n+2}}\binom{2n}{n},$$
    d'où :
    $$I=-\dfrac{\pi}{2}\sum_{n=0}^{+\infty}\dfrac{(-1)^n}{2^{4n}(2n+1)^2}\binom{2n}{n}.$$
    Ça ressemble au développement en série entière de $\operatorname{argsh}$. On a :
    $$\operatorname{argsh}t=\sum_{n=0}^{+\infty}\dfrac{(-1)^nt^{2n+1}}{2^{2n}(2n+1)}\binom{2n}{n}.$$
    donc :
    $$I=-\pi\int_0^{1/2}\frac{\operatorname{argsh} t}{t}dt.$$
    En posant $t=\operatorname{sh}x$ j'obtiens :
    $$\int_0^{1/2}\frac{\operatorname{argsh} t}{t}dt= \int_0^{\ln\varphi}x\coth x\,dx$$
    où $\varphi=\dfrac{1+\sqrt{5}}{2}$ est le nombre d'or.

    J'utilise maintenant l'égalité
    $$\coth x=1+2\dfrac{e^{-2x}}{1-e^{-2x}}=1+2\sum_{n=1}^{+\infty}e^{-2nx}$$
    et j'intervertis $\sum$ et $\int$ :
    $$\int_0^{\ln\varphi}x\coth x\,dx=\int_0^{\ln\varphi}x\,dx+2\sum_{n=1}^{+\infty}\int_0^{\ln\varphi}x e^{-2nx}\,dx$$
    qui donne en intégrant par parties :
    $$\int_0^{\ln\varphi}x\coth x\,dx=\dfrac{\ln^2\varphi}{2}-\ln\phi\sum_{n=1}^{+\infty}\dfrac{1}{n\varphi^{2n}}-\dfrac{1}{2}\sum_{n=1}^{+\infty}\dfrac{1}{n^2\varphi^{2n}}+\dfrac{1}{2}\sum_{n=1}^{+\infty}\dfrac{1}{n^2}.$$
    Or $\displaystyle\sum_{n=1}^{+\infty}\dfrac{1}{n\varphi^{2n}}=-\ln(1-1/\varphi^2)=\ln\varphi$, $\displaystyle\sum_{n=1}^{+\infty}\dfrac{1}{n^2\varphi^{2n}}=\operatorname{Li}_2(1/\varphi^2)=\dfrac{\pi^2}{15}-\ln^2\varphi$ et $\displaystyle\sum_{n=1}^{+\infty}\dfrac{1}{n^2}=\dfrac{\pi^2}{6}$, donc :
    $$\int_0^{\ln\varphi}x\coth x\,dx=\dfrac{\pi^2}{20}$$
    et finalement :
    $$I=-\dfrac{\pi^3}{20}.$$
  • Bravo !

    A partir de,

    $\displaystyle I=-2\int_0^{1/2}\dfrac{\phi(y)}{y\sqrt{1-4y^2}}dy$

    On peut faire plus simple pour la suite me semble-t-il.

    L'intégrale $\displaystyle \int_0^1 \frac{\arctan(tx)}{x\sqrt{1-x^2}}dx$ est calculable sauf erreur.

    A la fin on se retrouve à évaluer me semble-t-il,

    $\displaystyle \int_0^1 \dfrac{\text{arcsinh}\left(\frac{x}{2}\right)}{x}dx$
  • Oui, je me doutais qu'on pouvait faire plus simple. Peux-tu détailler tes calculs ?
  • Hébus:

    Les calculs sont plus compliqués que je pensais en écrivant mon dernier message.
    On peut toutefois éviter le recours à des développements en série.

    Soit à calculer,

    $\displaystyle J=\int_0^1 \dfrac{\arctan\left(\frac{x}{2}\right)}{x}\ln\left(\frac{1-\sqrt{1-x^2}}{1+\sqrt{1-x^2}}\right)\,\text{dx}$

    On définit pour tout $x$ réel,

    $\displaystyle R(x)=\int_0^x \dfrac{\arctan\left(\frac{t}{2}\right)}{t}\text{dt}=\int_0^1 \dfrac{\arctan\left(\frac{tx}{2}\right)}{t}\text{dt}$

    On effectue une intégration par parties,

    $\begin{align}
    J&=\left[R(x)\ln\left(\frac{1-\sqrt{1-x^2}}{1+\sqrt{1-x^2}}\right)\right]_0^1-2\int_0^1 \int_0^1 \dfrac{\arctan\left(\frac{tx}{2}\right)}{tx\sqrt{1-x^2}}\text{dt}\text{dx}\\
    &=-2\int_0^1 \int_0^1 \dfrac{\arctan\left(\frac{tx}{2}\right)}{tx\sqrt{1-x^2}}\text{dt}\text{dx}
    \end{align}$

    On définit pour tout $t$ réel positif,

    $\displaystyle F(t)=\int_0^1 \dfrac{\arctan\left(tx \right)}{x\sqrt{1-x^2}}\text{dx}$

    Remarquer que $F(0)=0$.

    Pour tout $t$ réel positif,

    $\begin{align}
    F^\prime (t)&=\int_0^1 \dfrac{1}{(1+t^2x^2)\sqrt{1-x^2}}\text{dx}\\
    &=\frac{1}{\sqrt{1+t^2}}\left[-\arctan\left(\frac{\sqrt{1-x^2}}{x\sqrt{1+t^2}}\right)\right]_0^1\\
    &=\frac{\pi}{2\sqrt{1+t^2}}
    \end{align}$

    Donc pour tout $t$ réel positif,

    $\begin{align}
    F(t)&=\dfrac{\pi}{2}\text{arcsinh}(t)\\
    &=\dfrac{\pi}{2}\ln\left(t+\sqrt{1+t^2}\right)
    \end{align}$

    Ainsi,

    $\begin{align} J&=-2\int_0^1 \frac{F\left(\frac{t}{2}\right)}{t}\text{dt}\\
    &=-\pi\int_0^1 \frac{\ln\left(\frac{t}{2}+\sqrt{1+\frac{t^2}{4}}\right)}{t}\text{dt}
    \end{align}$

    On pose $\varphi=\frac{1+\sqrt{5}}{2}$ et on effectue le changement de variable $x=\frac{t}{2}+\sqrt{1+\frac{t^2}{4}}$,

    $\begin{align}
    J&=-\pi\int_1^{\varphi} \frac{\left(1+\frac{1}{x^2}\right)\ln x}{x-\frac{1}{x}}\,\text{dx}\\
    &=\pi\int_1^{\varphi}\frac{(1+x^2)\ln x}{x(1-x)(1+x)}\,\text{dx}\\
    &=\pi\int_1^{\varphi} \frac{\ln x}{x}\,\text{dx}+\pi\int_1^{\varphi} \frac{\ln x}{1-x}\,\text{dx}-\pi\int_1^{\varphi} \frac{\ln x}{1+x}\,\text{dx}\\
    &=\frac{1}{2}\pi \Big[\ln^2 x\Big]_1^{\varphi}+\pi\int_1^{\varphi} \frac{\ln x}{1-x}\,\text{dx}-\pi\int_1^{\varphi} \frac{\ln x}{1+x}\,\text{dx}\\
    &=\frac{1}{2}\pi\ln^2\varphi +\pi\int_1^{\varphi} \frac{\ln x}{1-x}\,\text{dx}-\pi\int_1^{\varphi} \frac{\ln x}{1+x}\,\text{dx}
    \end{align}$

    On pose,

    $\displaystyle A=\int_1^{\varphi} \frac{\ln x}{1+x}\,\text{dx}$


    $\displaystyle B=\int_1^{\varphi} \frac{\ln x}{1-x}\,\text{dx}$

    $\displaystyle K=\int_0^1 \frac{\ln(1+x)}{x}\,\text{dx}$

    $\displaystyle L=\int_0^1 \frac{\ln x}{1+x}\,\text{dx}$

    On effectue une intégration par parties,

    $\begin{align}
    L&=\Big[\ln(1+x)\ln x\Big]_0^1-\int_0^1 \frac{\ln(1+x)}{x}\,\text{dx}\\
    &=-\int_0^1 \frac{\ln(1+x)}{x}\,\text{dx}\\
    &=-K
    \end{align}$

    $\begin{align} B-A&=\int_1^{\varphi}\frac{2x\ln x}{1-x^2}\,\text{dx}
    \end{align}$

    On remarque que $\varphi=\varphi^2-1$ et on effectue le changement de variable $y=x^2-1$,

    $\begin{align} B-A&=-\frac{1}{2}\int_0^{\varphi} \frac{\ln(1+x)}{x}\,\text{dx}\\
    &=-\frac{1}{2}K-\frac{1}{2}\int_1^{\varphi} \frac{\ln(1+x)}{x}\,\text{dx}\\
    \end{align}$

    On remarque que $\varphi^2=\varphi+1$ et on effectue une intégration par parties,

    $\begin{align} B-A&=-\frac{1}{2}K-\frac{1}{2}\Big[\ln x\ln(1+x)\Big]_1^{\varphi}+\frac{1}{2}\int_1^{\varphi} \frac{\ln x}{1+x}\,\text{dx}\\
    &=-\frac{1}{2}K-\ln^2\varphi+\frac{1}{2}A
    \end{align}$

    Ainsi,

    $\begin{align} \frac{3}{2}A-B&=\frac{1}{2}K+\ln^2\varphi\\
    &=\ln^2\varphi-\frac{1}{2}L
    \end{align}$


    Remarquer que $1+\frac{1+\varphi}{1+\varphi}=\frac{2}{\varphi^2}$
    En effectuant le changement de variable $y=\frac{1-x}{1+x}$ dans l'intégrale suivante,

    $\begin{align} L+A&=\int_0^{\varphi} \frac{\ln x}{1+x}\,\text{dx}\\
    &=\int_{\frac{1-\varphi}{1+\varphi}}^1\frac{\ln\left(\frac{1-x}{1+x}\right)}{1+x}\,\text{dx}\\
    &=\int_{\frac{1-\varphi}{1+\varphi}}^1\frac{\ln(1-x)}{1+x}\,\text{dx}-\int_{\frac{1-\varphi}{1+\varphi}}^1\frac{\ln(1+x)}{1+x}\,\text{dx}\\
    &=\int_{\frac{1-\varphi}{1+\varphi}}^1\frac{\ln(1-x)}{1+x}\,\text{dx}-\dfrac{1}{2}\Big[\ln(1+x)^2\Big]_{\frac{1-\varphi}{1+\varphi}}^1\\
    &=\int_{\frac{1-\varphi}{1+\varphi}}^1\frac{\ln(1-x)}{1+x}\,\text{dx}-\dfrac{1}{2}\ln^2 2+\dfrac{1}{2}\ln^2\left(\frac{2}{\varphi^2}\right)
    \end{align}$

    On pose,

    $\displaystyle M=\int_0^1 \frac{\ln x}{1-x}\,\text{dx}$

    Remarquer que $1-\frac{1-\varphi}{1+\varphi}=\frac{2}{\varphi}$, $1-\frac{1}{\varphi}=\frac{1}{\varphi^2}$ et on effectue le changement de variable $y=\frac{1-x}{2}$,

    $\begin{align} L+A&=\int_0^\frac{1}{\varphi} \frac{\ln(2x)}{1-x}\,\text{dx}-\dfrac{1}{2}\ln^2 2+\dfrac{1}{2}\ln^2\left(\frac{2}{\varphi^2}\right)\\
    &=\int_0^\frac{1}{\varphi} \frac{\ln x}{1-x}\,\text{dx}+\int_0^\frac{1}{\varphi} \frac{\ln 2 }{1-x}\,\text{dx}-\dfrac{1}{2}\ln^2 2+\dfrac{1}{2}\ln^2\left(\frac{2}{\varphi^2}\right)\\
    &=-\int_\frac{1}{\varphi}^1 \frac{\ln x}{1-x}\,\text{dx}+\int_0^1 \frac{\ln x}{1-x}\,\text{dx}-\ln 2\Big[\ln(1-x)\Big]_0^{\frac{1}{\varphi}}-\dfrac{1}{2}\ln^2 2+\dfrac{1}{2}\ln^2\left(\frac{2}{\varphi^2}\right)\\
    &=-\int_\frac{1}{\varphi}^1 \frac{\ln x}{1-x}\,\text{dx}+M+2\ln 2\ln\varphi-\dfrac{1}{2}\ln^2 2+\dfrac{1}{2}\ln^2\left(\frac{2}{\varphi^2}\right)\\
    \end{align}$

    On effectue le changement de variable $y=\frac{1}{x}$,

    $\begin{align} L+A&=\int_1^\varphi \frac{\ln x}{x(x-1)}\,\text{dx}+M+2\ln 2\ln\varphi-\dfrac{1}{2}\ln^2 2+\dfrac{1}{2}\ln^2\left(\frac{2}{\varphi^2}\right)\\
    &=-\int_1^\varphi \frac{\ln x}{1-x}\,\text{dx}-\int_1^\varphi \frac{\ln x}{x}\,\text{dx}+M+2\ln 2\ln\varphi-\dfrac{1}{2}\ln^2 2+\dfrac{1}{2}\ln^2\left(\frac{2}{\varphi^2}\right)\\
    &=-B-\dfrac{1}{2}\Big[\ln^2 x\Big]_1^\varphi+M+2\ln 2\ln\varphi-\dfrac{1}{2}\ln^2 2+\dfrac{1}{2}\ln^2\left(\frac{2}{\varphi^2}\right)\\
    &=-B-\dfrac{1}{2}\ln^2\varphi+M+2\ln 2\ln\varphi-\dfrac{1}{2}\ln^2 2+\dfrac{1}{2}\ln^2\left(\frac{2}{\varphi^2}\right)\\
    &=-B+M+\frac{3}{2}\ln^2 \varphi
    \end{align}$

    Ainsi,

    $\displaystyle A+B=M-L+\frac{3}{2}\ln^2 \varphi$

    Or,

    $\begin{align}M-L&=\int_0^1 \frac{2x\ln x}{1-x^2}\,\text{dx}
    \end{align}$

    On effectue le changement de variable $y=x^2$,

    $\begin{align}M-L&=\int_0^1 \frac{\ln\left(\sqrt{x}\right)}{1-x}\,\text{dx}\\
    &=\frac{1}{2}\int_0^1 \frac{\ln x}{1-x}\,\text{dx}\\
    &=\frac{1}{2}M
    \end{align}$

    Donc,

    $\displaystyle L=\frac{1}{2}M$

    Ainsi,

    $\begin{cases}
    \frac{3}{2}A-B&=\ln^2\varphi-\frac{1}{4}M\\
    A+B&=\frac{1}{2}M+\frac{3}{2}\ln^2 \varphi
    \end{cases}$

    Donc,

    $\displaystyle A=\frac{1}{10}M+\ln^2 \varphi$
    $\displaystyle B=\frac{2}{5}M+\frac{1}{2}\ln^2 \varphi$

    Et ainsi,

    $\displaystyle J=\dfrac{3}{10}\pi M$

    Or, il est bien connu que $M=-\dfrac{\pi^2}{6}$, donc,

    $\boxed{\displaystyle J=-\frac{\pi^3}{20}}$
  • Une intégrale double que je ne sais pas (encore) calculer.
    La valeur donnée a été trouvée empiriquement (lindep...) et est très probablement correcte.
    Peut-être qu'il y a une façon pas trop compliquée de démontrer que cette forme close est la bonne.

    $\displaystyle \int_0^{\frac{\pi}{2}}\int_0^{\frac{\pi}{2}} \sin x \arctan^2(\sin x \sin y) \ dx \ dy\overset{?}{=}\frac{1}{32}\pi^3+\frac{1}{8}\pi^2\ln 2-\frac{1}{2}\text{G}\pi$

    $G$ est la constante de Catalan.
  • Bonjour @Fin de partie,

    Pour l'intégrale $\displaystyle I=\int_{0}^{{\pi\over 2}} \int_{0}^{{\pi\over 2}} \sin x \, arctan^2 (\sin x \sin y) dx dy$, on commence par démontrer l'existence, puis on change les variables $(x,y) \leadsto (u,v)$ avec $u=\sin x \sin y, v=\sin x$ et alors $\displaystyle I=\int_{0}^{1} \int_{u}^{1} du dv {arctan^2 u \over \sqrt{1-v^2} \sqrt{1-({u \over v})^2}} = \int_{0}^{1} du \, arctan^2 u \int_{u}^{1} {dv \over \sqrt{1-v^2} \sqrt{1-({u \over v})^2}} ={\pi\over 2} \int_{0}^{1} du \, arctan^2 u $ avec $\displaystyle \int_{u}^{1} {dv \over \sqrt{1-v^2} \sqrt{1-({u \over v})^2}} ={\pi\over 2} $ - qui s'obtient avec le changement de variables $\displaystyle v \leadsto \theta$ avec $\displaystyle v^2 = u^2+(1-u^2) \sin^2 \theta, \theta \in [0, {\pi \over 2}]$ - et $\displaystyle \int_{0}^{1} du \, arctan^2 u ={\pi^2 \over 16} + {\pi \over 4} \ln 2-G$ - qui s'obtient par deux intégration par parties : $\displaystyle \int_{0}^{1} du \, arctan^2 u = u \, arctan^2 u \mid_{0}^{1} - \int_{0}^{1} du {2u \over 1+u^2} arctan u$, $\displaystyle \int_{0}^{1} du {2u \over 1+u^2 } arctan u= \ln(1+u^2) arctan u \mid_{0}^{1} - \int_{0}^{1} du {\ln(1+u^2)\over 1+u^2}$ - qui termine la démonstration et confirme ton résultat.
Connectez-vous ou Inscrivez-vous pour répondre.
Success message!